47 votos

¿Es cada uno de $\int_0^\infty\frac{dx}{x^x},\int_0^\infty\frac{dx}{x^{x^{x^x}}},\int_0^\infty\frac{dx}{x^{x^{x^{x^{x^x}}}}},\cdots$ menos de $2$ ?

Hace unos años pregunté sobre la desigualdad Demostrar que $\int_0^\infty\frac1{x^x}\, dx<2$ . Cuando volví a revisarlo, encontré que cada una de las siguientes integrales de tetración $$\int_0^\infty\frac{dx}{x^x},\int_0^\infty\frac{dx}{x^{x^{x^x}}},\int_0^\infty\frac{dx}{x^{x^{x^{x^{x^x}}}}},\cdots$$ parecía estar delimitada por encima por $2$ . En el gráfico siguiente, cada índice representa la mitad del número de tetraciones.

Obviamente, un método de ataque consiste en demostrar que si $f_1(x)=x^x$ y $f_{k+1}(x)=x^{x^{f_k(x)}}$ entonces

  1. $\int_0^\infty dx/f_{k+1}(x)>\int_0^\infty dx/f_k(x)$ para cada $k>1$ y

  2. $\lim_{k\to\infty}\int_0^\infty dx/f_k(x)<2$ .

Observaciones.

  • El primer paso del método anterior significa que el área ganada en el intervalo $(0,1)$ es mayor que el área perdida en $(1,\infty)$ . Parece que las diferencias consecutivas (representadas a continuación como %97 ) $$\int_0^\infty\frac{dx}{f_{k+1}(x)}-\int_0^\infty\frac{dx}{f_k(x)}$$ del orden de $k^{-\log k}$ y también disminuyen monotónicamente en la mayoría de los casos.

He publicado este problema de forma cruzada <a href="https://mathoverflow.net/questions/417026/is-int-0-inftydx-over-xxxx-int-0-inftydx-over-xxxxxx">en MathOverflow </a>.

  • Para el segundo paso, el caso límite resulta muy fácil de demostrar. Tenemos $$\lim_{k\to\infty}\int_0^\infty\frac{dx}{f_k(x)}<\lim_{k\to\infty}\int_0^\infty\frac{dx}{g_k(x)}=-\int_0^\infty t\cdot\frac d{dt}\frac1{t^t}\,dt=\int_0^\infty\frac1{x^x}\,dx<2$$ donde $g_{k+1}(x)=x^{g_k(x)}$ y $g_1(x)=x$ .

9voto

River Li Puntos 101

Algunas reflexiones :

(No es una prueba rigurosa. Las integrales se calculan con Maple. Para una demostración rigurosa, necesitamos límites superiores analíticos para las integrales. )

Sea $$I(n) = \int_0^\infty \frac{1}{^{2n}x}\mathrm{d} x.$$

Se puede demostrar que $I(1) < 2$ y $I(2) < 2$ .

A continuación, supongamos que $n \ge 3$ .

Tenemos $$I_1(n) := \int_{\mathrm{e}^{-\mathrm{e}}}^1 \frac{1}{^{2n}x} \mathrm{d} x \le \int_{\mathrm{e}^{-\mathrm{e}}}^1 \frac{-\ln x}{W(-\ln x)} \mathrm{d} x < 1.495. $$

Tenemos $$I_2(n) := \int_1^{5/3} \frac{1}{^{2n}x}\mathrm{d} x \le \int_1^{5/3} \frac{1}{^{6}x}\mathrm{d} x < 0.385.$$

Desde $^4 x > 5$ para todos $x \ge 5/3$ tenemos $$I_3(n) := \int_{5/3}^\infty \frac{1}{^{2n}x}\mathrm{d} x \le \int_{5/3}^\infty \frac{1}{x^{x^5}}\mathrm{d} x < 0.00005.$$

Desde $^{2n}x \ge \mathrm{e}^{-1}$ para todos $0 < x < \mathrm{e}^{-\mathrm{e}}$ tenemos $$I_4(n) := \int_{\frac35\mathrm{e}^{-\mathrm{e}}}^{\mathrm{e}^{-\mathrm{e}}} \frac{1}{^{2n}x}\mathrm{d} x \le \int_{\frac35\mathrm{e}^{-\mathrm{e}}}^{\mathrm{e}^{-\mathrm{e}}} \frac{1}{x^{x^{1/\mathrm{e}}}}\mathrm{d} x < 0.0715.$$

Se puede utilizar la inducción matemática para demostrar que $^{2n} x \ge \frac34 $ para todos $0 < x < \frac35 \mathrm{e}^{-\mathrm{e}}$ . Tenemos $$I_5(n) := \int_0^{\frac35 \mathrm{e}^{-\mathrm{e}}} \frac{1}{^{2n}x}\mathrm{d} x \le \int_0^{\frac35 \mathrm{e}^{-\mathrm{e}}} \frac{1}{x^{x^{3/4}}}\mathrm{d} x < 0.0485.$$

Así, $I(n) = I_1(n) + I_2(n) + I_3(n) + I_4(n) + I_5(n) < 2$ .

4voto

Stefan Näwe Puntos 1728

En la región $0<x<1$ , $\frac{1}{^{2n}x}$ es una secuencia creciente. Si $y\leq e$ y $x<1$ entonces $$\frac{1}{y}\geq\frac{1}{e}\quad\Rightarrow \quad x^{1/y}\leq x^{1/e}\quad\Rightarrow \quad x^{x^{1/y}}\geq x^{x^{1/e}}\geq\frac{1}{e}\quad\Rightarrow \quad\frac{1}{x^{x^{1/y}}}\leq e$$ por lo que la secuencia está acotada anteriormente por $e$ . Por el teorema de convergencia monótona, la secuencia debe converger a alguna $y\leq e$ y tal $y$ debe cumplir $$\frac{1}{y}=x^{x^{1/y}}$$ Cuando, $x\geq e^{-e}$ la solución viene dada simplemente por $x=\frac{1}{^2y}$ . Sin embargo, cuando $x<e^{-e}$ debemos usar la paramaterización que dio Thomas Browning: $$(x,y)=(t^{(t^{-t/(1-t)})/(1-t)},t^{-t/(1-t)}) \quad 0<t<1$$ De ello se deduce que $$\int_0^1\frac{1}{^{2n}x}dx<1+\int_1^e\frac{1}{^2x}dx-\int_0^1t^{(t^{-t/(1-t)})/(1-t)}\frac{d}{dt}t^{-t/(1-t)}dt<1+0.6734-0.0904=1.583$$ Para $n\geq 3$ obtenemos $$\int_1^\infty\frac{1}{^{2n}x}dx\leq \int_1^\infty\frac{1}{^{6}x}dx=\int_0^1\frac{(1/x)^2}{^{6}(1/x)}dx<0.3838$$ Por lo tanto, $$\int_0^\infty\frac{1}{^{2n}x}dx \leq 1.583+0.3838=1.9668<2$$ Como ya sabemos $$\int_0^\infty \frac{1}{^2x}<2$$ sólo tenemos que comprobar $$\int_0^\infty \frac{1}{^4x}dx=\int_0^1 \frac{1}{^4x}dx+\int_0^1\frac{(1/x)^2}{^{4}(1/x)}dx<1.4026+0.4324=1.835<2$$ lo que completa la prueba.

Comentario . No hay mucha elegancia integral. Los límites pueden calcularse utilizando sumas de Reimann con mallas suficientemente pequeñas ( $\delta\approx10^{-6}$ ) para acotar el término de error. Ampliando la término de error dado aquí a funciones muchos a uno, encontramos que las integrales calculadas aquí tienen un término de error dado por $$k\delta(\max f-\min f)\quad\quad\quad k=\max\{|f^{-1}(y)|:y\in\mathbb{R}\}$$ donde $f$ es el integrando, y los máximos y mínimos están en el rango de integración. Para las funciones anteriores $f\geq 0$ , $k\leq 2$ y los límites de los máximos son fáciles de calcular.

-1voto

Rob Corless Puntos 13

Creo que se puede hacer (dice incautamente). Hace tiempo reproduje los pasos de Euler con esta iteración; quizá pueda hacerlo de nuevo. El comportamiento global no es, si no recuerdo mal, demasiado malo. En el intervalo $\exp(\exp(-1))< x < \infty$ la iteración $a_{n+1} = x^{a_n}$ diverge, creciendo monotónicamente. En el intervalo $\exp(-\exp(1)) \leq x \leq \exp(\exp(-1))$ la iteración converge (de nuevo creo que monotónicamente). En el intervalo $0 < x < \exp(-\exp(1))$ los iterados pares convergen (monotónicamente) a una respuesta, mientras que los iterados Impares convergen a otra (la iteración tiene dos ciclos, pero además la convergencia es monotónica en cada subsecuencia).

Así que espere que se podría demostrar la afirmación dividiendo el intervalo en esos tres y considerando cada uno por separado.

Ahora voy a intentar poner mi dinero donde está mi boca.

i-Ciencias.com

I-Ciencias es una comunidad de estudiantes y amantes de la ciencia en la que puedes resolver tus problemas y dudas.
Puedes consultar las preguntas de otros usuarios, hacer tus propias preguntas o resolver las de los demás.

Powered by:

X